Overunity.com Archives is Temporarily on Read Mode Only!



Free Energy will change the World - Free Energy will stop Climate Change - Free Energy will give us hope
and we will not surrender until free energy will be enabled all over the world, to power planes, cars, ships and trains.
Free energy will help the poor to become independent of needing expensive fuels.
So all in all Free energy will bring far more peace to the world than any other invention has already brought to the world.
Those beautiful words were written by Stefan Hartmann/Owner/Admin at overunity.com
Unfortunately now, Stefan Hartmann is very ill and He needs our help
Stefan wanted that I have all these massive data to get it back online
even being as ill as Stefan is, he transferred all databases and folders
that without his help, this Forum Archives would have never been published here
so, please, as the Webmaster and Creator of this Forum, I am asking that you help him
by making a donation on the Paypal Button above
Thanks to ALL for your help!!


A possible violation of the Law of Conservation of Energy

Started by Zetetic, April 14, 2015, 04:59:57 PM

Previous topic - Next topic

0 Members and 1 Guest are viewing this topic.

ayeaye

It is always A. In my experiment, the upper magnet in A is the stator magnet, and the lower magnet in A is one of the rotor magnets. The stator magnet is a kind of long, so only one pole, the north pole, really matters, the effect of the other pole is very small and can be disregarded. On my drawing, the upper magnet in A is shown as moving, and the lower magnet is shown as standing still.

I used a long cylindrical neodymium magnet as the stator magnet. It is important in that experiment that the stator magnet is strong, and the rotor magnets are small. Mainly because the rotor is lighter then.

So the upper magnet is really the stator magnet, but it is a kind of opposite on the drawing, i showed it so because i thought that it's better to explain it that way. As it doesn't really matter which one of the magnets stands still and which one moves, because it only matters that the magnets move relative to each other.

Zetetic



ayeaye,



Okay.  Your magnets are always orientated like in drawing A in reply #4.  That makes sense.  Thank you for clearing that up for me.

I now have a second basic question about your drawing.  Hopefully we can also clear that up for me and then I can finally get to understanding and then addressing the point you are making with your drawing.

In your drawing you show the moving magnet before reaching the fixed magnet and then you show the moving magnet after passing the fixed magnet.  And, when the magnets are in those positons, you indicate that there is "no repulsion."

I think I get your point, but I think the logic needs to be fleshed out a little bit more.

When the magnets are in position X (in the drawing below) there is no repulsion (so I agree with your drawing about that), but there is attraction (which is not indicated in your drawing).  And this attraction inhibits the leftward movement of the moving magnet.

And when the magnets are in position Y (in the drawing below) there is no "repulsion" in the sense of pushing the magnet rightward.  However, when the magnets are in position Y there is both attraction pulling the magnet leftward and (I believe) repulsion pushing the magnet leftward.

This is counterintuitive.  It seems as if when the moving magnet is to the right of the fixed magnet that the moving magnet should be repulsed to the right.  However, the magnetic lines of flux, obviously, extend out beyond the physical location of actual magnet itself.  And so, when the moving magnet is just slightly to the right of the fixed magnet there are repulsive magnetic lines of flux to the right of the moving magnet and this will push the moving magnet leftward.  It counterintuitive, but if you think about it it makes sense, and it's one of those weird aspects of magnetic interaction that I find fascinating.

But if the magnets are in position Z (in the drawing below), if the moving magnet is just slightly more to the right of the fixed magnet, then there is repulsion which will push the moving magnet rightward (and this is not represented in your drawing).

This can be seen in your video "Field lines chain motor 1/4" at about 1 minute and 21 seconds.  You place the fixed magnet at the start slightly more to the right than in the other iterations and the moving magnets (in this case it is the other magnets that are the moveable) are repulsed away.

So, again, I'm trying to understand the basics of your drawing before trying to understand the point you are making with it.

There is no repulsion at point X (as you indicate in your drawing), but there is also attraction (which is not included in your drawing).

There is no "repulsion" at point Y (as you indicate in your drawing) in the sense of pushing the moving magnet rightwards (but there is a leftward repulsion).

And there is repulsion when the moving magnet is to the right of the fixed magnet, point Z, but this occurs when the moving magnet is slightly more to the right of the fixed magnet than point Y (and this is not indicated in your drawing).

No?

Did I get something wrong?

Let me know!

Again, it's a cool design.  Thanks for sharing it!


- Zet

(PS:  I agree with you that it doesn't matter which is the fixed magnet and which is the moving magnet (a difference between your videos and your drawing) but only relative the orientation between the two matters.)



ayeaye

The pole of the magnet moves by field lines. What concerns the repulsion in the beginning and the attraction in the end, then this happens only when the field lines go from a pole of the standing (lower) magnet to the pole of the moving magnet. This is insignificant when the lower magnet is a part of a chain of magnets. Then the field lines from its south pole go to the north pole of the previous magnet, and the field lines from its north pole go to the south pole of the next magnet.

This drawing was mostly drawn with the field lines chain design in mind. With a stand alone magnet some field lines certainly go to the pole of the moving (upper) magnet. Then it may just depend on how long are the field lines. Whether there is any overunity in that case, i don't know, this drawing was to explain the results of the field lines chain experiment.

ayeaye

So the concept basically is, that the magnetic field is an asymmetric field, so in some circumstances it can do work. And my experiments seem to confirm this. Yet it cannot provide permanent motion at least in a mechanical way.

OK, tell about your thing, what is the reason why it can be overunity?

Zetetic



I thought I was getting a better handle on your proposal, but now I'm not so sure.  I feel like maybe I've taken one step forward and two steps back (in trying to understand it).

I'm trying!

Based on what you wrote in Reply #7 I have yet another basic question.

There is a concept in the physics of magnetics called the "unmagnetized" state.  "Unmagnetized" is different from "demagnetized."  Here is a link to a description of the former (if you're interested):

http://www.irm.umn.edu/hg2m/hg2m_d/hg2m_d.html

It's somewhat technical, but I think it may be relevant to what you are working on.

When a magnet is "demagnetized" the poles of the particles within it become randomized and so there is no longer an external magnetic field and thus the effects of the field are gone.  And when a magnet becomes "unmagnetized" (as a ferromagnet has the natural tendency to do) the magnet moves to a state with more and more multiple poles within the single body, and this reigns in the external magnetic field and so here also the effects of the external field are also (essentially) gone.

I've never heard of someone making a ring of magnets in order to reign in the external magnetic field.  But in your Reply #7 it seems like you might be suggesting that that is what you are doing.  And, again while I've never heard of someone doing this, it make sense that your ring of magnets would essentially lead to a state of "unmagnetization" (in the sense of the word used in the link above).

But, if this is what you are suggesting, then I'm confused.  This would then mean the "unmagnetized" ring would not significantly interact with the external fixed in place magnet.  And isn't the interaction between the ring of magnets and the external fixed magnet the whole point of your design?

Again, I'm sorry.  I'm trying!



---



Okay, back to the idea in my original post.  Thanks for asking!

The link in the original post does not lead to an argument for an Overunity Device but rather to an argument for why the Law of Conservation of Energy is false.  And the two things, while different, are related.  And I can explain this in very simple terms.

The Law of Conservation of Energy says that while energy can change forms the total amount of energy remains the same.

There are different kinds of energy.  Motion is a form of energy ("kinetic energy").  Heat is a form of energy ("thermal energy").  A ball some distance above the surface of the Earth is a form of energy ("potential energy").  And so on.

This "Law" says that energy can change from one form to another (e.g. motion can become heat and heat can become motion) but the amount of energy remains the same.

So, for example, say I roll a ball across the floor.  When the ball leaves my hand it is in motion, and this motion is a certain amount of energy, and let's say its 10 units of kinetic energy.  As the ball rolls across the floor it eventually comes to a stop.  The 10 units of kinetic energy is now gone.  What brings the ball to a stop is the friction between the ball and the floor.  And friction generates heat.  And so, when the ball comes to a stop, there is a loss of 10 units of kinetic energy but there is also an equal increase of 10 units of thermal energy (heat).

And so, in the ball rolling across the floor example, energy has changed forms (motion becomes heat) but the total amount of energy remains the same (10 units of kinetic energy becomes 10 units of thermal energy).



Okay, why is this relevant to OU?

Say someone was successful at making a working device that once set in motion continued moving forever (such as if you were able to get "Field lines chain motor" to spin forever).  There is a certain amount of energy put into the system at the start.  It takes energy for your hand to originally set it into motion (there is a decrease in biological energy and an equal increase in kinetic energy in the form of the spinning disk).  Now, say, it keeps moving.  This would mean there is no decrease in kinetic energy.  But, as it moves there is friction (between the spinning disk and the bearings and the axel and between the spinning disk and the surrounding air and so on).  And friction generates heat (thermal energy).

So, if your design (or anyone else's in this forum) was to keep moving after being set into motion, despite encountering friction, there would be no decrease in kinetic energy while all the time it is moving more and more thermal energy would be generated.  Energy would be created!  The total amount of energy within the Universe would increase!

And this would be a violation of the Law of Conservation of Energy.  The total amount of energy would not remain the same.  There would be no decrease in kinetic energy while there is an increase thermal energy.

(There is also the quest for a perpetual motion machine of the second kind which does not violate the Law of Conservation of Energy but rather violates the Second Law of Thermodynamics, but that's a different topic.)



So, back to my original post (and the argument it links to).

I've spent years playing around with trying to build a perpetual motion machine (free energy device, overunity device).  But I've also spent years just playing around with the logic of the Law of Conservation of Energy itself.

And I believe I've been able to craft an argument that shows a violation of the Law of Conservation of Energy, which means, if I'm right, shows the Law of Conservation of Energy is false.

Unfortunately, the specific violation (and thus disproof) of this "Law" that I found does not also (I don't think) lead to the mechanics needed to design and build an OU device.

But, if I'm right, and I have shown this "Law" to be false, this would validate the efforts of you and me and everyone else in this forum.  If you were to show your "Field lines chain motor" to an actual physicist, he or she wouldn't waste any time looking at the mechanics of it to see if you've got something, they would "know" that there must be a flaw in there somewhere because the Law of Conservation of Energy tells them that such a device as yours could never work and so they'd just dismiss it out of hand without any examination.

If I'm right and I've found a flaw in (and thus disproved) the Law of Conservation of Energy in my argument (linked to in the OP), and if this could become established and well known, then efforts like yours in your design would be given credibility in the "established" scientific community.

And that's it.

That's the point of my post here.

If you read my argument in the link in the original post, let me know what you think!

(However, if you want to talk more about the mechanics of your design I'm happy to do so ... I would enjoy doing so!)



- Zet


(PS:  Oops.  I promised in my Reply #2 to explain the Law of Conservation of Energy and its relation to OU in a couple of short paragraphs.  Oops.  I guess I went a little bit overboard here.  Sorry.)